Will choose brainliest, i need number 4!

Will Choose Brainliest, I Need Number 4!

Answers

Answer 1

Answer:Show all the calculation if it is a numerical question

Step-by-step explanation:


Related Questions

Pre-Algebra
Due urgently
Tysm

Answers

Answer:

1. x = 5    2. y = 4      3. x = -2

Step-by-step explanation:

2x - y = 9             -5x + 2y = 13       3x + 4y = -6

4x + y = 21            5x + y = -1         7x(-1) + 4y(-1) = -14(-1)   <(multiply all by -1)

6x = 30                      3y = 12        -4x = 8

6       6                        3       3          4       4

x = 5                            y = 4              x = -2

I don’t know the answer I just need points

What is the volume of this rectangular prism?

Enter your answer in the box.

__cm³

Answers

I have an answer, its 56, that's because one side is 7, one side is 4 one side is 2 multiply and get 56.

7x9=56 which is your answer

please help me i have a brain fart i forgot

Answers

First your going to want to make either the x or the y have the same number in both equations. The easiest way to do that in this equation is by multiplying all of the bottom equation by 3. Then you subtract the equation to cancel the y variable and leave only the x. Once you’ve found x, put it back into either of the original equations to find y. Hope this helps :)

Answer:

The correct answer is -2,8

Step-by-step explanation

One month there were 6984 visitors at Metro Zoo. Of these visitors, 1561 had discount entrance vouchers. How many visitors paid regular price?

Answers

Answer:

5423 visitors paid regular price.

explanation:

visitors who paid regular price = total visitor - visitor with vouchers

→ 6984 - 1561

→ 5423

Answer:

5423

Step-by-step explanation:

can someone help me with this question?

Answers

Answer:

I think it would be B

Step-by-step explanation:

can i please get help step by step

Answers

Answer:

Answer: 18

Step-by-step explanation:

Follow the rule of BIDMAS -

(7 - 4)² = 3² = 9

9 x 2 = 18

(a-b)^2 * 2 = (7-4)^2 * 2 = 3^2 *2 = 9 * 2 =

18

(based of 'order of operation', look at diagram for help)

Hope that helps!

After a recent survey of 1485 people aged 18 to 24 showing that 78% were impressed with the special effects on a newly released movie, a statistician decided to test a claim by the film company stating that more than 80% of all people aged 18 to 24 who went to see the movie were impressed with the special effects. Using a 0.05 significance level, find the P-value and determine an initial conclusion about the claim. A. 0.0268; reject the null hypothesis B. 0.9732; fail to reject the null hypothesis C. 0.0268; fail to reject the null hypothesis D. 0.9732; reject the null hypothesis

Answers

(B Fail to reject the null hypothesis. The​ P-value is greater than the level of significance.

What is the null hypothesis?

The null hypothesis is a common arithmetic theory that asserts that no statistical relationship or significance exists between two sets of observed data and measured phenomena in a set of given, single, observed variables.

The null hypothesis is rejected, if the P-value is less than the level of significance.

Here, it is provided that the test is left-tailed and the z score is -1.23.

From, the table of the negative scores, the P-value for the left-tailed test is 0.1093.

Since the P-value (0.1093) is greater than the level of significance (0.05). The null hypothesis is not rejected.

Hence, the correct option is (B), Failing to reject the null hypothesis. The​ P-value is greater than the level of significance.

To know more about the null hypothesis follow

https://brainly.com/question/15980493

#SPJ2

Help me plsssss!!! also pls explain so I could understand <3

Answers

Answer: 6 is to 10, 39 is to 65, and 96 is to 160.

Step-by-step explanation: 3 * 1.6667 = 5, and 12 * 1.6667 = 20, therefore, all must be scaled to 1.6667. If you multiply the numbers to 1.6667, you will get the answers above.

Answer:

10,65,160

Step-by-step explanation:

You multiply everything by 5/3, since 3 × 5/3 is 5.

The graph of △ABC has coordinates A(−3, −1), B(−4, −4), and C(−1, −2)

Answers

Answer:  

Step-by-step explanation:

Isn’t it almost like the a b c d e f g h I j k l m n o p q r s t u v w x w z

Math question. giving brainliest!!!
what is 153 + 96?

Answers

Answer:

249

Step-by-step explanation:

153 + 96 = 249

The answer is 249

Solution:

1
153
+96
——-
249


153+96= 249

HELP
Write a verbal expression for 5(h + 7)

Answers

Answer:

5h+35

Step-by-step explanation:

i multiply by using the Distributive Property

Answer:

five times the sum of a number and seven.

hope it helps!

-50 + 5/13p when p= -26

Answers

Answer:

-60

Step-by-step explanation:

16905/338 hope it helps

help me pls I'll give brainliest to the person with the correct answer Which figure can be formed from the next ​

Answers

Answer:

The bottom one on the left

Step-by-step explanation:

I hope you have a great day!

PLS help will choose brainliest!

Answers

Step-by-step explanation:

Explained in the picture

Explain any mistakes in the renaming of the fractions below. Show the correct renaming.
1/4=3/12 2/3=6/12

Answers

Answer:

2/3 = 8/12

Step-by-step explanation:

1/4 is equal to 3/12, because 1/4 times 3/3 (which is 1) equals to 3/12.

2/3 is not equal to 6/12, because 6/12 can be simplified to 1/2. 2/3 is greater  than 1/2, not equal to half. So, 2/3 times 4/4 (which is 1) equals to 8/12.

Hope this helps :)

Have a nice day!

help to choose the one answer the question

Answers

Answer:

Answer 1

Step-by-step explanation:

28 per hour; x number of hours

-14 per week

378 after paying transportation

28x-14=378

Hope this helps.

Answer 1 & 2 are both right

Please help and I’ll give you brainliest.

Answers

Step-by-step explanation:

Tree A: y = 8x+16

because it was 16 when planted and grew at a pace of 8 inches/ year

Tree B: y = 2x+40

because it was 40 when planted and grew at a pace of 2 inches per year

Hope this helps!!!

**URGENT**
you dont have to show work

40. an ounce of prevention is said to be worth a pound of cure.
a. how many ounces of cure is 2.5 ounces of prevention worth?

b. How many ounces of prevention are worth 72 ounces of cure?

Answers

Answer:

a. 2.5 b.72

Step-by-step explanation:

because its worth the same amount

Help me pls. (Giving brainliest) :))))

Answers

Answer:

(B) is the answer for your question,

and please brainliest

A regular polygon is an. -sided polygon in which the sides are all the same length and are symmetrically placed about a common center

Answer:

B)

Step-by-step explanation:

A regular polygon has sides of equal length (and interior angles of equal measure).

Therefore, the regular polygon is B) (it's called an octagon as it has 8 sides)

I really need help on this!

Answers

Answer:

Step-by-step explanation:

P 1. 22 72 151

P 2. 357 473

500

                                   473

400

                            357

300

200

                  151

100

          72

50

           

   22

10  

     1      2      3      4     5    

Area of a Circle (PLEASE HELP)

Answers

Answer: The formula is shown in the explanation
28.3
12.6
50.3
153.9
452.4
9.62
452.4
490.9
63.6
153.9
1520.5
2463
explanation: the formula for the area of a circle is pie radius squared. if you took the radius off all of these, then plugged it in your calculator then you would get the answer. if you need show your work, just look for the radius off of the problems and rewrite the formula but replace the R with the actual radius in the problem

20y+5 using distributive property

Answers

Answer:

5(4y + 1)

Step-by-step explanation:

20y + 5 ← factor out 5 from each term

= 5(4y + 1)

20y+5 using distributive property

If a, b, c, and d are four different digits from 1 to 9, inclusive, then what's the largest possible value for the decimal a.b + c.d?

Answers

Answer:

18.3

Step-by-step explanation:

All the digits must be different and you must use the four highest digits. Give the higher two digits a higher place value. 9.7 + 8.6 = 18.3

Answer:

18.3

Step-by-step explanation:

help me pls I'll give brainliest to the person with the correct answer​

Answers

Answer:

i think the last

Step-by-step explanation:

if it were a survey umm i dont think it should be minutes because most likely they would answer 60. i dont think for sisters because they may know how many sisters he has. I dont think november because they may answer the same or may not know

Answer: I think the answer is "How many brothers do you have" because the other ones dont really fit as a statistical question to ask your friends on an essay,

Step-by-step explanation: I hope this helps if its wrong you can go ahead and delete my answer but i really hope its correct!

What can be determined about the sum of 8√ and 12.7?
Select words from the drop-down menus to correctly complete the sentences.

The value 8√ is
irrational/rational
and the value 12.7 is
irrational/rational
The sum of 8√ and 12.7 has a
non-repeating & non-terminating/repeating & -terminating
decimal expansion. Therefore, the sum of 8√ and 12.7is
irrational/rational

Answers

Answer: Let's look at the decimal number. We want to know where root 91 is in relation to the decimal number. The decimal number is between 12 and 13, or in other words, between and. Both of these numbers are larger than  meaning that the answer has to be A

Step-by-step explanation:

Which statement is true?

Answers

Answer:

the first answer

Step-by-step explanation:

you can see that for each starting number it is about 50% higher than the lower one, if it was 100% lower it would be too low

The first answer is true hop this helps …………

Can you please help I need help

Answers

Answer:

[tex]1.\ A=24\ ft^2[/tex]

[tex]2.\ A=36\ ft^2[/tex]

[tex]3.\ A=29.04\ cm^2[/tex]

[tex]4.\ A=1,624\ cm^2[/tex]

[tex]5.\ A=12.96\ cm^2[/tex]

[tex]6.\ A=28\ in^2[/tex]

What is the King of the Pencil Case?: The Ruler

Step-by-step explanation:

1. Surface area of a cube: [tex]6a^2\ a=edge[/tex]

[tex]A=6(2)^2\\A=6(4)\\A=24\ ft^2[/tex]

2. Lateral area of a rectangular prism: [tex]2 ( l + w ) h[/tex]

[tex]l=3\ ft\\w=1.5\ ft\\h=4\ ft[/tex]

[tex]A=2 ( l + w ) h\\A=2(3+1.5)4\\A=2*4=8\\A=8(3+1.5)\\A=8(4.5)\\A=36\ ft^2[/tex]

3. Surface area of a cube: [tex]6a^2\ a=edge[/tex]

[tex]A=6(2.2)^2\\A=6(4.84)\\A=29.04\ cm^2[/tex]

4. Surface area of a rectangular prism: [tex]2(lw + wh + hl)[/tex]

[tex]l=28\ cm\\w=14\ cm\\h=10\ cm[/tex]

[tex]A=2(lw) +2( wh) + 2(hl)\\A=2(28*14)+2(14*10)+2(10*28)\\A=784+280+560\\A=1,624\ cm^2[/tex]

5. Lateral area of a rectangular prism: [tex]2 ( l + w ) h[/tex]

[tex]l=1.8\ cm\\w=1.8\ cm\\h=1.8\ cm[/tex]

[tex]A=2 ( l + w ) h\\A=2(1.8+1.8)1.8\\A=2*1.8=3.6\\A=3.6(1.8+1.8)\\A=3.6(3.6)\\A=12.96\ cm^2[/tex]

6.  Surface area of a rectangular prism: [tex]2(lw + wh + hl)[/tex]

[tex]l=2\ cm\\w=1.5\ cm\\h=4\ cm[/tex]

[tex]A=2(lw) +2( wh) + 2(hl)\\A=2(1.5*2)+2(2*4)+2(4*1.5)\\A=6+16+12\\A=34\ in^2\\A=34-6\ [2(1.5*2)]\\A=28\ in^2[/tex]

Hope this helps!

1. A = 24 f12
2. A = 36 f12
3. A= 29.04 cm?
1. A = 1, 624 cm?
5. A = 12.96 cm?
6. A= 28 in?
What is the King of the Pencil Case?: The Ruler

Can u guys help me with this? ​

Answers

Answer:

5 cm

Step-by-step explanation:

9 - 4 = 5

the 9 is on the bottom line and the very top has a 4 so u just have to subtract

i hope this helps! have a wonderful rest of ur day :)

could u pls mark me as brainliest, i only need 1 more today and then ill rank up, if not then its ok! ty and let me know if you need anything else!!

its 5 because you subtract the 9-4

Find the total volume of the complex figure. Answer = in3 the same for the other one to

Answers

Answer:

31752in^3

Step-by-step explanation:

31752in^3. That’s the answer I need to fill in characters :)

4 was subtracted from a number and that difference was then divided by 19. after which, that quotient was multiplied by 5. the resulting product was -5. what was the number?
please show work and answer.

Answers

Answer:
-15

Explanation:
-5 / 5 = -1
-1 x 19 = -19
-19 + 4 = -15
Other Questions
An electric lightbulb in a lamp fixture is illuminating a small room. The system consists of the lightbulb, the lamp fixture, the air and the room.Which choice best describes the role of thermal energy in energy transformations? Alexa always gives a 15% tip when she eats at arestaurant. She gives a $2.13 tip. What was thecost of her meal? If you could work in any one of the three branches ofgovernment (Legislative, Executive, or Judicial) whichbranch would you choose?Include 3 reasons why you believe that branch would be the best towork in. Please could anyone help me????the picture below has it!!I need this fast, SO PLEASE HURRY. Please help me with this Given p | q and 24 = 130, find the measure of angle 1:P345 678970130HELP!! Determine the distance between points A (3, 5) and B (2, 1) rounded to the nearest tenth. What is it like having dissociative identity disorder?. Which of the following was a problem caused by trusts in the late 1800s?O A. Businesses spent most of their money on workers' salaries.B. Companies moved their factories to foreign countries.C. Consumers paid higher prices for goods and services.O OO D. Factories had a difficult time finding workers. Assume that x has a normal distribution with the specified mean and standard deviation. Find the indicated probability. (Round your answer to four decimal places.) = 8; = 6P(5 x 17) = Pythagorean theory . Thank you which of these functions is linear? The average distance from earth to the sun is 150,000,000 km what is the name of the unit scientists created that is equal to the distance from earth to the sun Lana wants to have $100,000 in 20 years. She plans to invest $8,000 to start and make yearly payments of $1,500 to the account. She will be receiving 7.5% interest compounded monthly on her investment. Will she reach her goal?NoYes En un edificio de cinco pisos viven las familias grados, sarabia, gutierrez, pardo y castro Which one of these is least like the others javascript ruby python mysql. PLEASE HELP ASAP!!!!!The measure of angle JKL can be represented using the expression 3x + 5. Point K has three lines extending from it. One line extends to point J, another to point M, and the other to point L. Angle J K M is 45 degrees. Angle M K L is x degrees.What is the degree measure of AngleJKL? What can multivariable calculus show for a data set. Drag the terms on the left to the appropriate blanks on the right to complete the sentences. Reset Help 1. A normal gene that, if mutated, can lead to cancer is called a(n) _______. 2. A gene that causes a cell to become cancerous is called a(n) _______. 3. A(n) ______ is a gene that encodes proteins that inhibit cell division. Such proteins normally help prevent cells from becoming cancerous. 4. A(n) ______ is a protein that promotes cell division. In excess, such a protein may lead to cancer. Read the excerpt from "Healthy Eating."But no one can ignore the damage being wrought by all this bad eating. More than one out of three adults in the United States -- 37.5 percent -- are obese, according to the Centers for Disease Control and Prevention. Some of the leading causes of death -- heart disease, stroke and type 2 diabetes -- are directly linked to obesity. And childhood obesity has tripled in the last 30 years. With more than a third of kids between 6 and 19 considered obese, the CDC reports, they are at substantially more risk for developing cardiovascular disease, diabetes, bone and joint conditions, and cancer.How does the author develop her argument in this part of the article? A She provides statistics to show how harmful a poor diet can be. B She gives several reasons why people should eat healthier foods. C She shares anecdotes to show why children need healthier diets. D She uses scientific facts to tell why eating habits have changed.